Der Virialsatz und ein Delta-Funktionspotential

Der Virialsatz sagt uns also:

2 T = R v .

Jetzt habe ich mich gefragt, was passieren würde, wenn V die Form hat:

v ( R R ' ) = v 0 δ ( D ) ( R R ' ) , Wo δ ( D ) ( R R ' ) ist die Delta-Funktion in D-Dimensionen. Ich bin mir nicht sicher warum, aber ich denke, dass ich das bekommen sollte:

R v = 1 D v 0 da das als Produkt verschiedener Komponenten ausgeschriebene Delta ist:

δ ( D ) ( R R ' ) = 1 D e T ( G ) ich = 1 D δ ( X ich X ich ' ) , mit X ich die verschiedenen Komponenten des Vektors R , angegeben in der Basis mit Metrik G, wobei D e T ( G ) gibt das D-dimensionale Volumenelement in der Basis an e ich .

Ich weiß nicht, ob es dafür eine strengere Begründung gibt? Oder ob das überhaupt stimmt?

Nachtrag: eine andere Perspektive:

Eine andere Möglichkeit, es zu betrachten, ist, wenn ich meinen Vektor neu skaliere R bij ein Faktor λ , Ich bekomme:

δ ( D ) ( λ R λ R ' ) = 1 λ D δ ( D ) ( R R ' ) . Das lässt mich auch denken, dass ich die obige Beziehung für den Virialsatz bekommen sollte. Aber ich bin mir meiner Argumentation immer noch nicht sicher!

Zusätzliche Nachfrage nach Potential (notwendig für endliches System)

Neben meinem Delta-Potential habe ich noch ein zusätzliches Begrenzungspotential, um die Teilchen zusammenzuhalten. Der Einfachheit halber nehme ich eine harmonische Falle v ( R ) = 1 2 M ω 2 R 2 was die Teilchen zusammenhält! Das ist also der andere Term des Potenzials, aber diesen habe ich in meiner Frage nicht berücksichtigt, weil er für meine Berechnungen kein Problem darstellte!

Ich bin mir nicht sicher, aber wenn Sie eine Ableitung der Delta-Funktion wünschen, müssen Sie Ihren Virialsatz auch über den Raum integrieren. Dann ist die rechte Seite bei partieller Integration gleich D v 0 .
@swish Für den quantenmechanischen Erwartungswert sollte ich tatsächlich über die Stelle mit irgendeiner Wellenfunktion integrieren. Vielleicht klappt das ja :).
Ich bin ein wenig eingerostet, aber interessante Frage. Für mich ergeben einige der Konstanten keinen Sinn. Das Problem ist, dass ich nicht glaube, dass die Ableitung der Delta-Funktion definiert ist, zumindest in dem Sinne, wie sie vom Gradienten verwendet wird. Zum Spaß versuchte ich eine verallgemeinerte zweite Ableitung, indem ich die Ableitung zweier parametrisierter Stufenfunktionen nahm (und sagte, das Delta sei ähnlich einer Grenze der Parametrisierung), und ich bekam Null. Wenn das Delta springt und in einem einzigen Moment um den gleichen Betrag zurückgeht, klingt Null nicht verrückt, wenn auch nicht besonders aussagekräftig.
@DerekE Ja, ich habe über den Kommentar von swish nachgedacht, und für Deltafunktionen kann ich die Tatsache verwenden, dass X ( D / D X ) δ ( X ) = δ ( X ) was durch partielle Integration bewiesen wird. Aber im klassischen Kontext (wo der Erwartungswert eine Integration über die Zeit ist) kann ich so eine Substitution nicht vornehmen, also stecke ich immer noch fest :(.
Das ist ordentlich, gefällt mir. :) Obwohl ich den Beweis immer noch nicht sehe, muss ich damit spielen. Bei Euler-Lagrange brauchte man eine unabhängige Variablenfunktion, um auf eine Äquivalenz unter dem Integral zu schließen. Ich werde damit herumspielen, hier einen zu finden. Interessantes Ergebnis, aber. Ich hatte fast etwas ähnliches beim Anschauen δ ( X ) = e 2 π ich X ξ D ξ , aber bei der Anwendung der partiellen Integration waren die Dinge an der Grenze.
@DerekE Ich habe keine Darstellung verwendet, nur die Tatsache, dass δ ( X ) wird nur beim Integrieren mit einer Testfunktion unter dem Integral definiert, und für QM integriere ich tatsächlich über x, wo dieses Ergebnis nützlich ist. Im klassischen Fall geht die Integration über die Zeit und deshalb stecke ich immer noch fest :(
@Nick Ich denke, meine Sorge ist dann das Schreiben ( D / D X ) δ ( X ) an erster Stelle.
@DerekE Die Ableitung einer Verteilung wird durch den Mechanismus der partiellen Integration ( en.wikipedia.org/wiki/Dirac_delta_function ) definiert.

Antworten (1)

Kommentare zur Frage (v8):

  1. Der Virialsatz gilt normalerweise für periodische oder begrenzte Systeme, aber paarweise attraktive Deltafunktionspotentiale würden kein begrenztes System darstellen, es sei denn, das System ist zusätzlich in einer Box eingeschlossen. (OP hat in einem Update (v9) der Frage ein zusätzliches Potenzial eingeführt, um die Partikel einzudämmen.)

  2. Wenn wir uns auf eine paarweise Wechselwirkung konzentrieren (von den vielen paarweisen Wechselwirkungen), dann das attraktive Delta-Funktionspotential

    (A) v ( R )   :=   A δ D ( R ) , A   >   0
    ist klassisch schlecht definiert und muss reguliert werden. Man könnte hoffen, dass die quantenmechanische Verschmierung der Wellenfunktion das Potential (A) wohldefiniert macht. Dies ist jedoch nicht möglich z D > 2 : Das Potential (A) ist quantenmechanisch nach unten unbeschränkt für D > 2 (Siehe zB diesen Phys.Se-Beitrag. Die begrenzende Dimension D = 2 Fall ist nur für hinreichend schwache attraktive Delta-Funktionspotentiale (A) nach unten begrenzt.)

Ja, natürlich müssen die Partikel eingeschlossen werden (das hätte ich wahrscheinlich erwähnen sollen), da sie sonst ins Unendliche schweben würden. Das Delta-Potential ist in der klassischen Mechanik als Hard-Sphere-Potential und in der Quantenmechanik als Näherung für das Atompotential durch den Mechanismus der niederenergetischen S-Wellen-Streuung zu sehen. Das sagst du also praktisch v kann nur in D = 2 berechnet werden ?
siehe auch meine Bearbeitung zu meiner Frage!